I found this interesting (to me, at least) when I was solving this problem:
"Let be positive reals so that . Prove that: ."
At first, my thought was: "Oh, so it's obvious that . But what about the ? There's no way to throw it away." And right now, I'm stuck at that exact point. Reverse Rearrangement doesn't seem to work, and there is a "seemingly" trivial solution using Buffalo Way, but I want to find a "better" way than just plug-and-bash.
So, my question is: Is there a "good-looking" way to solve the problem? If there is, what is your inspiration?
Any help would be greatly appreciated. Thanks for reading!
P/S: If you can find any counterexamples, please tell me immediately and I will delete/edit this note.
Easy Math Editor
This discussion board is a place to discuss our Daily Challenges and the math and science related to those challenges. Explanations are more than just a solution — they should explain the steps and thinking strategies that you used to obtain the solution. Comments should further the discussion of math and science.
When posting on Brilliant:
*italics*
or_italics_
**bold**
or__bold__
paragraph 1
paragraph 2
[example link](https://brilliant.org)
> This is a quote
\(
...\)
or\[
...\]
to ensure proper formatting.2 \times 3
2^{34}
a_{i-1}
\frac{2}{3}
\sqrt{2}
\sum_{i=1}^3
\sin \theta
\boxed{123}
Comments
If you've proven kn≥a1a2…an for any a1,a2,…,an,k, substitute k←k+b and ai←ai+b and you're done.
Another method: aim to maximize (a1+b)(a2+b)…(an+b). Prove that if ∑ai<nk then this product is not maximized. Prove that if there is any ai=aj then this product is also not maximized. So the maximum of the product is achieved when ai=k for all i, done.
Log in to reply
That is actually the one I've been thinking about: I can't guarantee that the substitution will fit. If I could then I think this problem would've been too easy as it's so trivial.
Log in to reply
It works. Prove the special case k′n≥a1′a2′…an′, and use it as a lemma.
By AM-GM, n(a1+b)(a2+b)⋯(an+b)≤n(a1+b)+(a2+b)+⋯+(an+b)=na1+a2+⋯+an+b≤k+b. Therefore, (a1+b)(a2+b)⋯(an+b)≤(k+b)n.
In other words, direct AM-GM works.
You might be thinking of another problem, namely ∏(ai+b)≤(n∏ai+b)n.
Log in to reply
Thanks for mentioning.
What is "Buffalo way"? Is this some new inequality trick?
Log in to reply
Here's the link.